Why is C wrong?
Why is C wrong?
Raheel 07:27PM
  • December 2001 LSAT
  • SEC1
  • Q14
1
Reply
General Principle vs. Argumentative
How do you tell the difference?
Raheel 07:25PM
  • June 1995 LSAT
  • SEC3
  • Q11
1
Reply
How would you eliminate E faster?
I was stuck between the correct answer and E, but picked the correct answer
Raheel 07:24PM
  • June 1994 LSAT
  • SEC4
  • Q24
1
Reply
Answer A
Could someone please provide another explanation? I'm having difficulty seeing how the answer ca...
yckim2180 Sunday at 12:52AM
  • December 1994 LSAT
  • SEC4
  • Q22
1
Reply
What am I missing?
The question asks, " the tax provides financial incentive to residents to do each of the followin...
Anthony-Resendes Sunday at 12:47AM
  • December 1994 LSAT
  • SEC4
  • Q21
1
Reply
Explanation
Hi can someone go over why the other answers were wrong? I want to make sure I understood why the...
Jasmin1 Sunday at 12:45AM
  • December 1994 LSAT
  • SEC4
  • Q17
1
Reply
What is this?
This does not seem like a weaken question. Never seen anything like this question. How often is t...
Tyler808 Sunday at 12:41AM
  • December 1994 LSAT
  • SEC4
  • Q17
1
Reply
Please help ASAP
Hi, please explain why A isn't a correct answer choice
Jason10 Saturday at 10:26PM
  • December 1994 LSAT
  • SEC2
  • Q22
1
Reply
I can't see any explanation for this question.
Is this a glitch? Also could please kindly explain why this answer is correct?
hassay18 Saturday at 03:09PM
  • October 1994 LSAT
  • SEC3
  • Q19
1
Reply
Why is A better than B?
Why is A better than B? I don't understand how B is not right. Shouldn't it be the conclusion -> ...
Raheel Saturday at 02:39PM
  • December 2002 LSAT
  • SEC4
  • Q14
1
Reply
How many meta-structures are there?
And where can I learn more about these meta-structures? Is there a list available somewhere?
hassay18 Saturday at 02:28PM
  • October 1994 LSAT
  • SEC3
  • Q13
1
Reply
answer explanations?
where are the answer explanations for these practice test?
francolby Saturday at 02:26PM
  • February 1992 LSAT
  • SEC2
  • Q1
1
Reply
Why is A wrong?
I was between A and E for this question, and last second changed from A to E. I can't quite reali...
nathanielpfitzpatrick Saturday at 02:23PM
  • December 1998 LSAT
  • SEC4
  • Q9
1
Reply
How do you get to B?
A lot of the answer choices seemed right to me, and I picked E because B seemed out of scope?
Raheel Saturday at 02:13PM
  • December 2000 LSAT
  • SEC3
  • Q24
1
Reply
Why is D wrong?
Why is D wrong?
Raheel Saturday at 02:10PM
  • June 2000 LSAT
  • SEC2
  • Q19
1
Reply
Why is A wrong?
Why is A wrong?
Raheel Saturday at 02:05PM
  • October 2000 LSAT
  • SEC4
  • Q9
1
Reply
Can you explain E?
Can you explain all the answers and why E is correct?
capoleway@gmail.com on July 16 at 09:58AM
  • December 1997 LSAT
  • SEC2
  • Q18
2
Replies
The answer explination did not help.
Could someone go more into why B is right?
Raheel on July 15 at 11:00PM
  • September 2019 LSAT
  • SEC2
  • Q16
1
Reply
Why not B?
Why not B?
Raheel on July 15 at 10:32PM
  • October 1991 LSAT
  • SEC2
  • Q3
1
Reply
Answer Anticipation Issue
My answer anticipation to this question was wrong. It was going along the lines of maybe the EKG ...
Raheel on July 15 at 02:52PM
  • October 2010 LSAT
  • SEC3
  • Q20
1
Reply